What is Limit: Definition and 1000 Discussions

In mathematics, the limit inferior and limit superior of a sequence can be thought of as limiting (i.e., eventual and extreme) bounds on the sequence. They can be thought of in a similar fashion for a function (see limit of a function). For a set, they are the infimum and supremum of the set's limit points, respectively. In general, when there are multiple objects around which a sequence, function, or set accumulates, the inferior and superior limits extract the smallest and largest of them; the type of object and the measure of size is context-dependent, but the notion of extreme limits is invariant.
Limit inferior is also called infimum limit, limit infimum, liminf, inferior limit, lower limit, or inner limit; limit superior is also known as supremum limit, limit supremum, limsup, superior limit, upper limit, or outer limit.

The limit inferior of a sequence




x

n




{\displaystyle x_{n}}
is denoted by





lim inf

n





x

n




or





lim
_



n






x

n


.


{\displaystyle \liminf _{n\to \infty }x_{n}\quad {\text{or}}\quad \varliminf _{n\to \infty }x_{n}.}
The limit superior of a sequence




x

n




{\displaystyle x_{n}}
is denoted by





lim sup

n





x

n




or





lim
¯



n






x

n


.


{\displaystyle \limsup _{n\to \infty }x_{n}\quad {\text{or}}\quad \varlimsup _{n\to \infty }x_{n}.}

View More On Wikipedia.org
  1. LtIvan

    I What is the y-limit of the inverse tangent function?

    Allo, When I was experimenting with graphing functions, I noticed the inverse tangent, or arctanget, curves away from y=2, or may be less. What is the y limit for the inverse tangent function? Does it for ever increase, or terminate at a co-ordinate?
  2. T

    MHB Does the Limit of the Function Approach Zero as (x, y) Tends to (0,0)?

    I need to find $$\lim_{{(x, y)}\to{(0,0)}} \frac{x^2 - y^2}{\sqrt{x^2 + y^2}}$$ If I plug in zero, I get an indeterminate form. How do I resolve the indeterminate form?
  3. T

    MHB Evaluating limit of 2 variable function

    I have $$\lim_{{(x, y)}\to{(0, 0)}} \frac{x^4 - y^4}{x^4 + x^2y^2 + y^4}$$ If I evaluate the limit along the x-axis, I get $$\lim_{{(x, y)}\to{(0, 0)}} \frac{x^4 - y^4}{x^4 + x^2y^2 + y^4}$$ which evaluates to $1$. If I evaluate the limit along the y-axis, I get $$\lim_{{y}\to{0}} \frac{...
  4. T

    MHB Evaluating limit limit of multivariable function

    I have $$\lim_{{(x, y)}\to{(0, 0)}} \frac{x}{x^2 + y^2}$$ We can approach the limit on the x-axis, so the values of $x$ will change and the values of $y$ will stay : $$\lim_{{x}\to{0}} \frac{x}{x^2}$$ I suppose I can take hospital's rule and get $$\lim_{{x}\to{0}} \frac{x}{x^2}$$...
  5. T

    MHB Exploring the Limit of $|x|^2$ as $n \to \infty$

    I have $$\lim_{{n}\to{\infty}} \frac{|x|^2}{(2n + 3)(2n + 2)}$$ I can see that for smaller values of $x$ the limit is 0, but what if $x$ equals infinity, wouldn't that be an indeterminate form?
  6. M

    B Definition of the limit of a sequence

    The definition of a limit of a sequence, if the limit is finite, is: lim n >infinity un (un is a sequence) = l <=> ∀ε> 0, ∃N: n > N => |un - l| < ε This just means that un for n > N has to be a number for which: l -ε < un < l + ε Now, I'm wondering, can't we just say: n > N => |un -l| <...
  7. GIM

    I QCD scale and massless limit of u & d quarks

    Hello! Could anybody help me? My wondering seems so trivial, but I can't skip it. They say that since u and d quarks are much lighter than QCD scale(~200MeV), in reality we can consider the QCD Lagrangian has an approximate global chiral symmetry with respect to these two flavors. At first, it...
  8. T

    MHB Limit Comparison Test: Does L Approaching Infinity Matter?

    The limit comparison test states that if $a_n$ and $b_n$ are both positive and $L = \lim_{{n}\to{\infty} } \frac{a_n}{b_n} > 0$ then $\sum_{}^{} a_n$ will converge if $\sum_{}^{} b_n$ and $\sum_{}^{} a_n$ will diverge if $\sum_{}^{} b_n$ diverges. Does this rule also apply if $L$ diverges to...
  9. T

    MHB Limit Function: Finding Solution

    I'm trying to find the limit of this function: $$\lim_{{n}\to{\infty}} \frac{n^n}{({n + 1})^{n + 1}}$$ I can simplify it to this: $$\lim_{{n}\to{\infty}} \frac{n^n}{({n + 1})^{n}(n + 1)}$$ But I'm not sure of the best way to proceed.
  10. Rectifier

    Limit of $\sqrt{x+1} - \sqrt{x}$ as $x \to \infty$

    The problem $$ \lim_{x \rightarrow \infty} \left( \sqrt{x+1} - \sqrt{x} \right) $$ The attempt ## \left( \sqrt{x+1} - \sqrt{x} \right) = \frac{\left( \sqrt{x+1} - \sqrt{x} \right)\left( \sqrt{x+1} + \sqrt{x} \right) }{\left( \sqrt{x+1} + \sqrt{x} \right) } = \frac{x+1 - x }{\left(...
  11. chwala

    Limit of a trigonometric function

    Homework Statement Mod note: Edited the following to fix the LaTeX[/B] compute ##\lim_{n \rightarrow +0} \frac {8-9cos x+cos 3x} {sin^4(2x)}####\lim_{n \rightarrow +\infty} \frac {\sin(x)} x## ##\lim_{n \rightarrow +\infty} \frac {\sin(x)} x##ok find limit as x→0 for the function ##[ 8-9cos x...
  12. Rectifier

    Solving an Infinite Limit: Should I Factor?

    The problem \lim_{x\rightarrow \infty} \frac{x^4 + x \ln x}{x + \left( \frac{2}{3} \right)^x} The attempt \lim_{x\rightarrow \infty} \frac{x^4 + x \ln x}{x + \left( \frac{2}{3} \right)^x} = \lim_{x\rightarrow \infty} \frac{x^4(1 + \frac{x \ln x}{x^4}) }{x + \left( \frac{2}{3} \right)^x}...
  13. T

    MHB Solving Series Limit Problem: Find Convergence/Divergence

    I have this limit: $$\sum_{k = 1}^{\infty} {(\frac{e }{3})}^{k}$$ Which method can I use to find if it converges or diverges?
  14. T

    MHB Evaluating a limit with a factorial

    We are starting sequences, and in one of the examples we have this limit: $$\lim_{{n}\to{\infty}} \frac{R^n}{n!}$$ We let $M$ equal a non-negative integer such that $ M \le R < M + 1$ I don't get the following step: For $n > M$, we write $Rn/n!$ as a product of n factors: $$\frac{R^n}{n!}...
  15. T

    MHB Limit of a Geometric Sequence: How to Evaluate?

    I have this limit: $$\lim_{{n}\to{\infty}} {(\frac{2}{3})}^{n}$$ I know the answer is 0 but how can I evaluate this?
  16. T

    MHB Evaluate Limit: $$\lim_{x\to\infty} (-1)^nn^3 + 2^{-n}$$

    I have this limit: $$\lim_{{x}\to{\infty}} {(-1)}^{n}{n}^{3} + {2}^{-n}$$ and I'm unsure how to evaluate it or how to apply L'hopital's rule to this limit.
  17. T

    MHB Finding Limit of $\ln$ Sequence

    If I have this sequence $$a_n = \ln\left({\frac{n}{n^2 + 1}}\right)$$ I need to find: $$ \lim_{{n}\to{\infty}} \ln\left({\frac{n}{n^2 + 1}}\right)$$ Shouldn't I be able to find the limit of$$ \lim_{{n}\to{\infty}} \frac{n}{n^2 + 1}$$ (which is $0$) and then substitute the result of that...
  18. T

    MHB Limit of Natural Log Sequence: How to Find It Using L'Hopital's Rule?

    I have this sequence: $${a}_{n} = \ln \left(\frac{12n + 2}{-9 + 4n}\right)$$ I need to find the limit of this sequence. How can I go about this? Do I need to apply L'Hopitals rule? I'm unsure how to simplify this expression. If I use the rule $\ln(\frac{a}{b}) = \ln a - \ln b$ I get $\infty -...
  19. P

    Why Do My Answers Differ on These Limit Problems?

    Hey! Can somebody take a look on these two limit problems? I don't agree with the answer to #15, which is supposed to be 0 while I get infinity. #16 seems to ask to find the value of the sum...I posted a pic of my attempts to solve the problems below. My attempts:
  20. MrDickinson

    I Can someone me simplify this expression....

    lim_(h->0^-) (e^(x+h)/((x+h)^2-1)-e^(x+h)/(x^2-1))/h = -(2 e^x x)/(x^2-1)^2 I know how to differentiate the expression using the quotient rule; however, I want to use the limit definition of a derivative to practice it more.This desire to practice led me into a trap! Now I just can't simplify...
  21. N

    Double integrals with variable upper limit

    How can you compute F(k) = k\int^{\infty}_{0}dy\int^{y}_{0}dx f(kx,y) in C. I know about Python's scipy.integrate.dblquad function but it's just too slow. I have written some Cython code with a 2D gaussian quadrature function in C but it only takes doubles as limits. I think C doesn't have...
  22. Tollendal

    A Negative Absolute Temperature: Defying Gravity?

    Dr. Achim Rosch, a theoretical physicist at the University of Cologne in Germany, who proposed the technique used by Dr. Ulrich Schneider and his team to create in laboratory negative absolute temperature, have calculated that whereas clouds of atoms would normally be pulled downwards by...
  23. T

    MHB This limit does not exist as it approaches 1 from both sides.

    If I have this limit: $$\lim_{{R}\to{1}} \frac{1}{R - 1}$$ I try to apply L'hopital's rule: The derivative of 1 is 0, and the derivative of $R - 1$ is 1. So I get $\frac{0}{1}$ which is 0. But apparently the answer is infinity. What am I doing wrong?
  24. JasMath33

    Help Solve Calculus Limit Proof Homework Statement

    Homework Statement I am posting this for another student who I noticed did not have the proof in the problem. Here is what she said. Let's try and help her out. I have been working on the problem below and I am stuck. I am stuck primarily because of the part where is says x=0. If x-0, it...
  25. H

    B Limit of Focal Spot Size for Lensing EM Radiation Across Frequencies

    Did not know how to word this properly. Looking for an equation to show the behaviour of how EM can be "lensed" as a function of wavelength/frequency. ie RF waves can not be lensed/focussed to a spot. what equation determines the minimum spot size a lens can focus EM radiation as a function...
  26. J

    Help with Epsilon Delta Proof of Multivariable Limit

    Homework Statement Hey guys. I am having a little trouble answering this question. I am teaching myself calc 3 and am a little confused here (and thus can't ask a teacher). I need to find the limit as (x,y) approaches (0,1) of f(x,y) when f(x,y)=(xy-x)/(x^2+y^2-2y+1). Homework Equations...
  27. U

    MHB Limit of = (sin nx) / (sin x) as n goes to infinity.

    Hello everyone. I need help trying to calculate/ trying to realize what the limit function of (sin nx)/(sin x) as n goes to infinity is. from another topic here on MBH ("Show δn = (sin nx) / (pi x) is a delta distribution") and after research with Wolfram Alpha I know that the limit function...
  28. ChloeYip

    B Choosing Epsilon for a Limit Problem: A High School Student's Guide

    I have only encountered questions that f(x)-L that can be factorize to get a constant, and delta is epsilon divide that number, as a high school student. I have no idea how to choose a epsilon for this question. Thanks.
  29. JBC

    I Bayard Alpert Gauge X-ray limit

    When the cathode emits electrons which are accelerated towards the grid, usually on its way it will ionise a molecule in the vacuum. However at a certain low pressure there are too few molecules and therefore the electron will hit the grid and emit an x-ray. My question is, wouldn't electrons...
  30. Jezza

    How Can the Limit of (ln(1+x))^x as x Approaches 0 Be Evaluated Correctly?

    Homework Statement \lim\limits_{x \to 0} \left(\ln(1+x)\right)^x Homework Equations Maclaurin series: \ln(1+x) = x - \frac{x^2}{2} + \frac{x^3}{3} + ... + (-1)^{r+1} \frac{x^r}{r} + ... The Attempt at a Solution We're considering vanishingly small x, so just taking the first term in the...
  31. T

    I Proving the limit of sinx/x as x approaches 0 equals 1

    this is a geometric proof from James Stewart's calculus textbook page 192. I'm confused in the sequence of inequalities as part of the proof... theta = arcAB < AB + EB ==> arcAB < AE + ED. How did EB turned into ED? please check the picture I've uploaded with this post
  32. ecoo

    Proving the Limit in a Power Series: How Does the Solution Work?

    Homework Statement Section is on using power series to calculate functions, the problem is on proving the limit, solution is also attached but I do not see how the solution proves the limit. Homework Equations Convergent power series form The Attempt at a Solution I attempted to represent...
  33. DaTario

    I Prime Number Theorem: the meaning of the limit

    Hi All. I have a doubt concerning the limit: $$ \lim_{n \to \infty} \frac{\pi (n)}{Li(n)} = 1 $$. This mathematical statement does not imply that both functions converge to the same value. The main reason is that both tend to infinity as n tend to infinity. I would like to ask you if it is...
  34. Matejxx1

    Limit problem ( can't find the mistake)

    Homework Statement Hi. My professor asked me if I know to solve this limit and I tried doing it, however I didn't get the same answer as him. Question: What is the limit of (cos(x)*cos(2x)*cos(3x)*...*cos(nx)-1)/x^2 as x approches 0 Homework Equations / The Attempt at a Solution [/B] So to...
  35. Elnur Hajiyev

    A Are Killing Horizon and Stationary Limit Surface the same?

    I know that Killing horizon is the hypersurface on which timelike Killing vector field becomes null. Beyond that surface Killing vector field becomes spacelike. But Stationary Limit Surface has also such a property. I wonder, if they are the same thing, if so, why is there different names for...
  36. terryds

    Solve Limit Problem: Find a+b for ax+b=25

    Homework Statement If a and b satisfy ##\lim_{x->0}\frac{\sqrt{ax+b}-5}{x} = \frac{1}{2}##, then a+b equals... A. -15 B. -5 C. 5 D. 15 E. 30 Homework Equations L'hospitalThe Attempt at a Solution By using L'hospital, I get b=a^2 Then, I got stuck.. Substituting b=a^2 into the limit...
  37. lep11

    Solving tricky limit (involving defined integral and sine)

    Homework Statement Calculate ##\lim_{x\to 1} \frac{1}{x-1} \int_{1}^{f(x)} sin(\pi t^2) dt##. f is differentiable in the neighbourhood of point ##x=1## and ##f(1)=1##. Homework Equations If ##f## is continuous on a closed interval ##[a,b]##, then there exists ##ξ∈]a,b[## such that...
  38. terryds

    Solving a Trigonometric Limit Problem

    Homework Statement ##\lim_{a\rightarrow b} \frac{tan\ a - tan\ b}{1+(1-\frac{a}{b})\ tan\ a\ tan\ b - \frac{a}{b}}## = ...Homework Equations tan (a - b) = (tan a - tan b)/(1+tan a tan b) The Attempt at a Solution [/B] I don't know how to convert it to the form of tan (a-b) since there are...
  39. P

    In the limit as A --> ∞, what does the function become?

    Homework Statement The function is fA(x) = A, |x| < 1/A, and 0, |x| > 1/A Homework Equations δ(x) = ∞, x=1, and 0 otherwise The Attempt at a Solution I think the answer is the Dirac delta function, however I noticed that if you integrate fA(x) between -∞ and ∞ you get 2, which if I remember...
  40. I

    Limit points, closure of set (Is my proof correct?)

    Homework Statement Let ##E'## be the set of all limit points of a set ##E##. Prove that ##E'## is closed. Prove that ##E## and ##\bar E = E \cup E'## have the same limit points. Do ##E## and ##E'## always have the same limit points? Homework Equations Theorem: (i) ##\bar E## is closed (ii)...
  41. ebos

    I Speed of light or Speed limit of Spacetime?

    I'm having a difficult time researching the answer to my question about the speed of light. Now obviously it is a speed not only reserved for light but also all other massless particles/waves. It's obviously a constant property of our Spacetime since we can manipulate th speeds of different...
  42. Alpharup

    I Proving limit theorems when limit tends to infinity

    Am using Spivak and he defines limit of a function f 1. As it approaches a point a. 2.As it approaches infinity. He also defines limit f(x)=∞ x->a But though in solving exercises, we can see that all the three definitions are consistent with each other, I am not...
  43. P

    MHB Is the Limit of x/x! as x Approaches 0 Equal to 0 or Does it Not Exist?

    An interesting question has been posted by Brilliant.org. What is $\displaystyle \begin{align*} \lim_{x \to 0} \frac{x}{x!} \end{align*}$?My intuition tells me that the limit does not exist. My reasons for this are: 1. A limit can only exist if its left hand and right hand limits exist and...
  44. S

    I Boltzmann distribution for spin-1/2 dipole: high T limit

    The analysis of the distribution of spins for a paramagnetic solid in a B field shows that the probability of a dipole being aligned/anti-aligned with the B field ##\to 0.5## as ##T \to \infty##. The intuitive justifications that I've read say that this is "expected" as thermal motion tends to...
  45. lep11

    How Do You Evaluate the Limit of (1/tan(x) - 1/x)/x Using Taylor Series?

    Homework Statement Evaluate the limit ##\lim_{x\to0} \frac{1}{x}(\frac{1}{tanx}-\frac{1}{x}) ## using Taylor's formula. (Hint: ##\frac{1}{1+c}=\frac{1-c^2+c^2}{1+c} ## may be useful) The Attempt at a Solution I began by substituting ##tanx## with ##x+\frac{x^3}{3}+x^3ε(x)##, where ε tends to...
  46. marellasunny

    Linear actuator limit switch replacement/repair

    Linear actuator spec.: 400mm stroke, 6000N, 2 limit switches, 5mm/sec, 12V DC supply, BIBUS IP65 casing. One of the limit switches of my LA (Linear Actuator) seems to have burned out. This happened because I accidentally plugged the LA leads into the AC mains socket (my LA is rated to run at...
  47. A

    MHB What is the limit of the difference of square roots of consecutive numbers?

    Hello, I am struggling to understand a simple question on limits. I have watched a video trying to explain the theory and even have the answer right in front of me but I still don't understand. Could somebody please explain the steps in detail for me just for the first question as I'm hoping...
  48. E

    Finding the Ordered Pairs for an Exponential Limit

    Homework Statement If ##a, b \in \{1,2,3,4,5,6\}##, then number of ordered pairs of ##(a,b)## such that ##\lim_{x\to0}{\left(\dfrac{a^x + b^x}{2}\right)}^{\frac{2}{x}} = 6## is Homework EquationsThe Attempt at a Solution So, this is a typical exponential limit...
  49. lep11

    Calculate limit of piecewise function

    Homework Statement a.) Let ##f,g:ℝ→ℝ## such that ##g(x)=sin x## and ##f(x)= \left\{ \begin{array}{ll} x^2, x∈ℚ \\ 0 , x∈ℝ\setminusℚ \\ \end{array} \right. ##. Calculate ##\lim_{x \rightarrow 0} \frac{f(x)}{g(x)}##. b.) Why l'Hospital rule cannot be applied here?The Attempt at a...
Back
Top